Brainliest giving to who gets this CORRECT first. Check all that apply.

Brainliest Giving To Who Gets This CORRECT First. Check All That Apply.

Answers

Answer 1

Answer:

D.

Step-by-step explanation:

a² -b²= (a-b)(a+b)

100x²-49y²= (10x-7y)(10x+7y)

Answer 2

Answer:

Answer is D

Step-by-step explanation:

The difference of the squares method is a short-cut method that is expressed a2 - b2 = ( a + b ) *(a-b). In this case, a2 and b2 should be perfect squares in which a and b are the square roots. In this case, 100 and 49 are perfect squares, hence the answer is D.


Related Questions

A company that manufactures laptop batteries claims the mean battery life is 16 hours. Assuming the distribution of battery life is approximately normal, a consumer group will conduct a hypothesis test to investigate whether the battery life is less than 16 hours. The group selected a random sample of 14 of the batteries and found an average life of 15.6 hours with a standard deviation of 0.8 hour.

Which of the following is the correct test statistic for the hypothesis test?

A. t=15.6−160.8
B. t=16−15.60.8
C. t=15.6−160.813
D. t=15.6−160.814
E. t=16−15.60.814

Answers

Answer:

The correct test statistic for the hypothesis test is [tex]t = -1.87[/tex]

Step-by-step explanation:

The null hypothesis is:

[tex]H_{0} = 16[/tex]

The alternate hypotesis is:

[tex]H_{1} < 16[/tex]

The test statistic is:

[tex]t = \frac{X - \mu}{\frac{s}{\sqrt{n}}}[/tex]

In which X is the sample mean, [tex]\mu[/tex] is the value tested at the null hypothesis, s is the standard deviation of the sample and n is the size of the sample.

In this question:

[tex]X = 15.6, \mu = 16, s = 0.8, n = 14[/tex]

So

[tex]t = \frac{X - \mu}{\frac{s}{\sqrt{n}}}[/tex]

[tex]t = \frac{15.6 - 16}{\frac{0.8}{\sqrt{14}}}[/tex]

[tex]t = -1.87[/tex]

The correct test statistic for the hypothesis test is [tex]t = -1.87[/tex]

Hypothesis test used to check the results of the experiments gives true for the meaningful results.. The correct test statistic for the hypothesis test is -1.871.

Given information-

The mean battery life of the laptop is 16 hours claimed by the company.

The random sample for the test is 14.

Average life of the batteries found out as 15.6 hours.

The deviation for this result is 0.8 hours.

What is hypothesis test?

Hypothesis test used to check the results of the experiments gives true for the meaningful results.

As the mean battery life of the laptop is 16 hours claimed by the company.The null hypothesis for the given problem is,

[tex]H_o\mu=16[/tex]

As average life of the batteries found out as 15.6 hours. Thus the alternate hypothesis for the given problem is,

[tex]H_1\mu<16[/tex]

One sample t test can be found using the below formula,

[tex]t=\dfrac{\overline x -\mu_o}{\dfrac{s}{\sqrt{n} } }[/tex]

Here, [tex]\overline x[/tex] is mean value, [tex]n[/tex] is the number of random sample and [tex]s[/tex] is the deviation.

Put the values,

[tex]t=\dfrac{15.6 -16}{\dfrac{0.8}{\sqrt{14} } }\\t=-1.871[/tex]

Thus the correct test statistic for the hypothesis test is -1.871.

Learn more about the hypothesis here;

https://brainly.com/question/2695653

Solve SA = πrs + π r2 for s A) -r B) SA + r C) SA π2r3 D) SA - πr2 πr

Answers

Answer:

(SA - π r^2)/πr = s

Step-by-step explanation:

SA = πrs + π r^2

Subtract  π r^2 from each side

SA - π r^2 = πrs + π r^2-π r^2

SA - π r^2 = πrs

Divide each side by  πr

(SA - π r^2)/πr = πrs/πr

(SA - π r^2)/πr = s

Answer:

D

Step-by-step explanation:

SA = πrs + πr²

SA - πr² = πrs

s = (SA - πr²)/πr

AB=
Round your answer to the nearest hundredth.
pleaseee

Answers

Answer:

[tex]c = \frac{2}{0.42} [/tex]

Step-by-step explanation:

AB = c

[tex] \frac{a}{sin \: A} = \frac{c}{sin \: C} \\ \frac{2}{sin \: 25} = \frac{c}{sin \: 90} \\ \frac{2}{0.42} = \frac{c}{1} \\ 0.42 \: c = 2 \\ c = \frac{2}{0.42} [/tex]

Answer:

4.73

Step-by-step explanation:

How far can a dog run into the woods?

Answers

Answer:

Half way

Step-by-step explanation:

Half way, because the dog can run all the way through the woods, but only half of the time he is going in, the rest of the time he is going out.

A data set with a mean of 34 and a standard deviation of 2.5 is normally distributed
According to the Empirical Rule, what percent of the data is in each of the following ranges? Round to the nearest tenth of a percent if necessary.
Between
34 and 39
Less than
31.5
Between
29 and 36.5
Percentage
%
%

Answers

Answer:

a) [tex] z= \frac{34-34}{2.5}= 0[/tex]

[tex] z= \frac{39-34}{2.5}= 2[/tex]

And we want the probability from 0 to two deviations above the mean and we got 95/2 = 47.5 %

b) [tex] P(X<31.5) [/tex]

[tex] z= \frac{31.5-34}{2.5}= -1[/tex]

So one deviation below the mean we have: (100-68)/2 = 16%

c) [tex] z= \frac{29-34}{2.5}= -2[/tex]

[tex] z= \frac{36.5-34}{2.5}= 1[/tex]

For this case below 2 deviation from the mean we have 2.5% and above 1 deviation from the mean we got 16% and then the percentage between -2 and 1 deviation above the mean we got: (100-16-2.5)% = 81.5%

Step-by-step explanation:

For this case we have a random variable with the following parameters:

[tex] X \sim N(\mu = 34, \sigma=2.5)[/tex]

From the empirical rule we know that within one deviation from the mean we have 68% of the values, within two deviations we have 95% and within 3 deviations we have 99.7% of the data.

We want to find the following probability:

[tex] P(34 < X<39)[/tex]

We can find the number of deviation from the mean with the z score formula:

[tex] z= \frac{X -\mu}{\sigma}[/tex]

And replacing we got

[tex] z= \frac{34-34}{2.5}= 0[/tex]

[tex] z= \frac{39-34}{2.5}= 2[/tex]

And we want the probability from 0 to two deviations above the mean and we got 95/2 = 47.5 %

For the second case:

[tex] P(X<31.5) [/tex]

[tex] z= \frac{31.5-34}{2.5}= -1[/tex]

So one deviation below the mean we have: (100-68)/2 = 16%

For the third case:

[tex] P(29 < X<36.5)[/tex]

And replacing we got:

[tex] z= \frac{29-34}{2.5}= -2[/tex]

[tex] z= \frac{36.5-34}{2.5}= 1[/tex]

For this case below 2 deviation from the mean we have 2.5% and above 1 deviation from the mean we got 16% and then the percentage between -2 and 1 deviation above the mean we got: (100-16-2.5)% = 81.5%

The complement of 20°17' is

Answers

Answer:69°43'

Step-by-step explanation:

Complementary angles add up to 90

Let them complement be y

y+20°17`=90°

Collect like terms

y=90-20°17' 20°17'=1217/60

y=90-1217/60

y=(60x90 -1 x 1217)/60

y=(5400-1217)/60

y=4183/60

y=69°43'

The altitude of an airplane is decreasing at a rate of 44 feet per second. What is the change in altitude of the airplane over a period of 34 seconds?

Answers

Answer:

1320 feet

Step-by-step explanation:

All we have to do is multiply the rate of change of altitude by the time it took the altitude to change.

The altitude of an airplane is decreasing at a rate of 44 feet per second. After 30 seconds, the change is altitude is:

44 * 30 = 1320 feet

The altitude of the airplane has changed by 1320 feet.

Giving brainliest for CORRECT awnser.

Answers

Answer:

64

Step-by-step explanation:

x^2 +16x+c

Take the coefficient of x

16

Divide by 2

16/2 =8

Square it

8^2 = 64

This is c

Answer:

c = 64

Step-by-step explanation:

The value for c is A. 64.  That comes from the process of completing the square where you take half the linear term, square it, and add it in.  Our linear term is 16.  Half of 16 is 8, and 8 squared is 64. 

What is the Surface Area of the figure below?




A
60 units2
B
60 units3
C
104 units2
D
104 units3

Answers

Answer:

D

Step-by-step explanation:

I'm really sry if it's wrong!

For the functions f(x)=6x−4 and g(x)=2x2+5, find (g∘f)(x).

Answers

Answer:

72x^2-96x+37

Step-by-step explanation:

Tia is planning a sailing party for her friends. The boat rental is $150 plus an
additional $15 per person. Tia has saved up $400 dollars. What is the
maximum number of people that can go sailing?
Identify the inequality to solve and the maximum number of people.

Answers

Answer:

16 people

Step-by-step explanation:

First subtract the cost of the rental from the amount of money:

$400-$150 = $250

Therefore Tia has $250 to spend for additional people. Then if each person is $15, divide the remaining amount of money by the amount of money per person:

$250/$15 = 16.67

Since you can't have 0.67 of a person she can have 16 people go with her.

This can also be modeled by this inequality:

[tex]150 + 15x \leqslant 400[/tex]

BRAINLIEST ASAP! LENGTH OF AC?

Answers

Answer:

2.33 units

Step-by-step explanation:

[tex]\tan 25\degree =\frac{AC}{5}\\\\0.46630 = \frac{AC}{5}\\\\AC = 0.46630 \times 5\\AC =2.3315\\AC = 2.33 \: units[/tex]

Two linear functions, f(x) and g(x), are combined by addition to form h(x). The same two linear functions are combined by multiplication to form j(x). Graphs of the resulting combined functions are shown. Which statements are true? Check all that apply. Graph A represents j(x). Graph A represents h(x). The y-intercepts for f(x) and g(x) can be 1 and 3. The y-intercepts for f(x) and g(x) can be 3 and 4. The rate of change of the sum of f(x) and g(x) is greater than that of either function.

Answers

Answer: this is the right answer Graph A represents j(x). The y-intercepts for f(x) and g(x) can be 1 and 3. And The rate of change of the sum of f(x) and g(x) is greater than that of either function. your welcome

Step-by-step explanation:

The rate of change of the sum of f(x) and g(x) is greater than that of either function.

How to determine the true statement?

From the question, we have the following parameters:

h(x) = f(x) + g(x)j(x) = f(x) * g(x)

The graphs of the functions are not given;

However, if all the functions are linear functions, then the slopes of the sum of functions f(x) and g(x) could be greater than the slopes of h(x) and j(x)

Hence, the true statement (by observation) is (c)

Read more about linear functions at:

https://brainly.com/question/4025726

#SPJ1

The Empire State Building weighs about 7.3×108pounds. The One World Trade Center building weighs about 88,200,000 pounds. What is the total weight, in pounds, of these two buildings? Expressing your answer in scientific notation in the form a×10b, what are the values of a and b?

Answers

Answer:

[tex]8.182X10^8 $pounds[/tex]

a=8.182 and b=8

Step-by-step explanation:

Weight of the Empire State Building =[tex]7.3X 10^8[/tex] pounds.

Weight of the One World Trade Center building= 88,200,000 pounds.

=[tex]8.82 X 10^7[/tex]

The addition of the two:

[tex]=7.3X 10^8+8.82 X 10^7\\$To make it easier to add, express both as powers of 8\\=7.3X 10^8+0.882 X 10^8\\=(7.3+0.882)X10^8\\=8.182X10^8 $ pounds[/tex]

Comparing with the form: [tex]aX10^b[/tex]

a=8.182 and b=8

Land's Bend sells a wide variety of outdoor equipment and clothing. The company sells both through mail order and via the internet. Random samples of sales receipts were studied for mail-order sales and internet sales, with the total purchase being recorded for each sale. A random sample of 7 sales receipts for mail-order sales results in a mean sale amount of $81.70 with a standard deviation of $18.75. A random sample of 11 sales receipts for internet sales results in a mean sale amount of $74.60 with a standard deviation of $28.25. Using this data, find the 80% confidence interval for the true mean difference between the mean amount of mail-order purchases and the mean amount of internet purchases. Assume that the population variances are not equal and that the two populations are normally distributed. Step 1 of 3 : Find the critical value that should be used in constructing the confidence interval. Round your answer to three decimal places.

Answers

Answer:

80% confidence interval for the true mean difference between the mean amount of mail-order purchases and the mean amount of internet purchases ([tex]\mu_1-\mu_2[/tex]) is [-9.132 , 23.332].

Step-by-step explanation:

We are given that a random sample of 7 sales receipts for mail-order sales results in a mean sale amount of $81.70 with a standard deviation of $18.75.

A random sample of 11 sales receipts for internet sales results in a mean sale amount of $74.60 with a standard deviation of $28.25.

Firstly, the Pivotal quantity for 80% confidence interval for the difference between population means is given by;

                            P.Q. =  [tex]\frac{(\bar X_1-\bar X_2)-(\mu_1-\mu_2)}{s_p\sqrt{\frac{1}{n_1} +\frac{1}{n_2} } }[/tex]  ~ [tex]t__n__1-_n__2-2[/tex]

where, [tex]\bar X_1[/tex] = sample mean sales receipts for mail-order sales = $81.70

[tex]\bar X_2[/tex] = sample mean sales receipts for internet sales = $74.60

[tex]s_1[/tex] = sample standard deviation for mail-order sales = $18.75

[tex]s_2[/tex] = sample standard deviation for internet sales = $28.25

[tex]n_1[/tex] = size of sales receipts for mail-order sales = 7

[tex]n_2[/tex] = size of sales receipts for internet sales = 11

Also, [tex]s_p=\sqrt{\frac{(n_1-1)s_1^{2} +(n_2-1)s_2^{2} }{n_1+n_2-2} }[/tex] = [tex]\sqrt{\frac{(7-1)\times 18.75^{2} +(11-1)\times 28.25^{2} }{7+11-2} }[/tex] = 25.11

Here for constructing 80% confidence interval we have used Two-sample t test statistics as we don't know about population standard deviations.

So, 80% confidence interval for the difference between population means, ([tex]\mu_1-\mu_2[/tex]) is ;

P(-1.337 < [tex]t_1_6[/tex] < 1.337) = 0.80  {As the critical value of t at 16 degree

                                         of freedom are -1.337 & 1.337 with P = 10%}  

P(-1.337 < [tex]\frac{(\bar X_1-\bar X_2)-(\mu_1-\mu_2)}{s_p\sqrt{\frac{1}{n_1} +\frac{1}{n_2} } }[/tex] < 1.337) = 0.80

P( [tex]-1.337 \times {s_p\sqrt{\frac{1}{n_1} +\frac{1}{n_2} } }[/tex] < [tex]{(\bar X_1-\bar X_2)-(\mu_1-\mu_2)}[/tex] < [tex]1.337 \times {s_p\sqrt{\frac{1}{n_1} +\frac{1}{n_2} } }[/tex] ) = 0.80

P( [tex](\bar X_1-\bar X_2)-1.337 \times {s_p\sqrt{\frac{1}{n_1} +\frac{1}{n_2} } }[/tex] < ([tex]\mu_1-\mu_2[/tex]) < [tex](\bar X_1-\bar X_2)+1.337 \times {s_p\sqrt{\frac{1}{n_1} +\frac{1}{n_2} } }[/tex] ) = 0.80

80% confidence interval for ([tex]\mu_1-\mu_2[/tex]) =

[ [tex](\bar X_1-\bar X_2)-1.337 \times {s_p\sqrt{\frac{1}{n_1} +\frac{1}{n_2} } }[/tex] , [tex](\bar X_1-\bar X_2)+1.337 \times {s_p\sqrt{\frac{1}{n_1} +\frac{1}{n_2} } }[/tex] ]

= [ [tex](81.70-74.60)-1.337 \times {25.11 \times \sqrt{\frac{1}{7} +\frac{1}{11} } }[/tex] , [tex](81.70-74.60)+1.337 \times {25.11 \times \sqrt{\frac{1}{7} +\frac{1}{11} } }[/tex] ]

= [-9.132 , 23.332]

Therefore, 80% confidence interval for the true mean difference between the mean amount of mail-order purchases and the mean amount of internet purchases ([tex]\mu_1-\mu_2[/tex]) is [-9.132 , 23.332].

PLEASE HELP!!! WILL MARK AS BRAINLIEST!!!

A colony of 300 bacteria doubles in size every 22 minutes can be represented by the exponential function y=300(2)x. If you want to know how many bacteria will be present about 66 minutes, what should you plug in for x?

Answers

Answer:

[tex] y = 300 (2)^x[/tex]

Where x represent the number of period of times of 22 minutes. If we want to know the value of the population after 66 minutes we need to find the value of x on this way:

[tex] x = 66 minutes *\frac{1period}{22 minutes}= 3[/tex]

So then we need to replace the value of x =3 and we got:

[tex] y= 300 (2)^3 = 2400[/tex]

Step-by-step explanation:

For this case we have the following function:

[tex] y = 300 (2)^x[/tex]

Where x represent the number of period of times of 22 minutes. If we want to know the value of the population after 66 minutes we need to find the value of x on this way:

[tex] x = 66 minutes *\frac{1period}{22 minutes}= 3[/tex]

So then we need to replace the value of x =3 and we got:

[tex] y= 300 (2)^3 = 2400[/tex]

100 POINTS
PLEASE PROVIDE STEPS
FIND FIRST DERIVATIVE AND SIMPLIFY ANSWER

Answers

Answer:

h'(x) = (-x² ln x + x² + 1) / (x (x² + 1)^(³/₂))

Step-by-step explanation:

h(x) = ln x / √(x² + 1)

You can either use quotient rule, or you can rewrite using negative exponents and use product rule.

h(x) = (ln x) (x² + 1)^(-½)

h'(x) = (ln x) (-½) (x² + 1)^(-³/₂) (2x) + (1/x) (x² + 1)^(-½)

h'(x) = (-x ln x) (x² + 1)^(-³/₂) + (1/x) (x² + 1)^(-½)

h'(x) = (x² + 1)^(-³/₂) (-x ln x + (1/x) (x² + 1))

h'(x) = (1/x) (x² + 1)^(-³/₂) (-x² ln x + x² + 1)

h'(x) = (-x² ln x + x² + 1) / (x (x² + 1)^(³/₂))

Solution:

h(x) = ln(x)/√x^2+1

h(x) = ln(x) * (x^2 + 1)^-1/2

h(x) = ln(x) * (-1/2) * (x^2 + 1)^-3/2 * 2x + 1/x * (x^2 + 1)^-1/2

h(x) = -x ln(x) * (x^2 + 1)^-3/2 + 1/x * (x^2 + 1)^-1/2

h(x) = (x^2 + 1)^-3/2 * (-x ln(x) + 1/x * (x^2 + 1))

h(x) = -x^2ln(x)+x^2+1/(x(x^2+1)^3/2)

Best of Luck!

What is the amplitude of y=3sin⁡(2x−1)+4?

Answers

Answer:

3

Step-by-step explanation:

Amplitude is the number in front of the sin or the number multiplied by the whole equation.  In this case its 3

solve for x
4x+5(5x-39)=153

Answers

Answer:

x=12

Step-by-step explanation:

4x+5 (5x-39) = 153

4x + 25x - 195 = 153

29x - 195 = 153

29x =  348

x = 12

Bob, Paula and Sam invest $50000 in a business. Bob invests $4000 more than Paul does and Paul invests $5000 more than Sam does. If the total profit was $70000, select the correct answer. Note that the profit is distributed proportionally based on the respective amount each invested. A. The ratio of the investment of Bob, Paula and Sam is 11:15:10. B. The ratio of the investment of Bob, Paula and Sam is 12:17:21. C. The ratio of the investment of Bob, Paula and Sam is 12:5:4. D. The profit of Paula was $23,800

Answers

Answer:

D

Step-by-step explanation:

since sam invest the least, let a be the amount invested by sam

sam = a

paul = a + 5000

bob = a + 5000 + 4000

3a + 14000 = 50000

3a = 36000

a = 12000

thus sam is 12000, paul is 17000 and Bob is 21000

therefore the ratio of B:P:S is 21:17:12

profit by paula is 17/50 x 70000 = 23800

The profit by Paula is 17/50 x 70000 = 23800.

We have given that Bob, Paula and Sam invest $50000 in a business. Bob invests $4000 more than Paul does and Paul invests $5000 more than Sam does. If the total profit was $70000

Since sam invest the least, let a be the amount invested by sam

Therefore we get,

sam = a

What is the investment of Paul?

The investment of Paul = a + 5000

Bob = a + 5000 + 4000

3a + 14000 = 50000

3a = 36000

divide both sides by 3 so we get,

a=36000/3

a = 12000

Therefore, sam is 12000,

paul =5000+12000=17000 and

Bob =12000+9000= 21000

Therefore the ratio of B:P:S is 21:17:12

The profit by Paula is 17/50 x 70000 = 23800.

To learn more about the investement visit:

https://brainly.com/question/25300925

Example on worksheet (subtracting by adding up)

50-29?

29+20=49

49+1=50

20+1=21

50-29=21

My question is where did the 20 come from? can you help me understand this?

Answers

Answer:

(49 - 29) + 1 =(50 - 29)

Step-by-step explanation:

The 20 comes from the subtraction of 29 in both sides of the previous step equality. In the following, I transcript the complete procedure and I add the step that you need to understand why 20 appears (in bold numbers):

29+20=49

49+1=50

(49 - 29) + 1 =(50 - 29)

20+1=21

50-29=21

hence, it was only nesseraty to subtract 29

Oil is leaking from an oil tanker, and an expanding circle of oil is spreading on the ocean. The radius, r, of

modeled by the function r(s)=315, where sis time in seconds.

The area of the spill when s=5 seconds is

1 square inches.

Reset

Reset

Next

Next

Answers

Answer:

[tex]45\pi$ square inches[/tex]

Step-by-step explanation:

The radius, r of the expanding circle of oil is modeled by the function:

[tex]r=3\sqrt{s}[/tex] , where s is time in seconds.

When s=5

Radius [tex]r(5)=3\sqrt{5}$ inches[/tex]

Area of a circle [tex]=\pi r^2[/tex]

Therefore, the area of the oil spill when s=5 seconds

[tex]=\pi* (3\sqrt{5})^2\\=45\pi$ square inches[/tex]

The area of the spill when s=5 seconds is 45pi square inches.

A movie theater decreased the size of its popcorn bags by 20%. If the old bags held 15 cups of popcorn, how much do the new bags hold

Answers

Answer:

Your answer will be [tex]12[/tex] cups of popcorn.

Step-by-step explanation:

To find out how much the new bags hold, you need to find out the discount.

[tex]\frac{20}{100 } = .2[/tex]

[tex]15 * .2 = 3[/tex]

We know that the discount is [tex]3[/tex].

To figure out how much the new bags hold, subtract by the old bags.

[tex]15 - 3 = 12[/tex]

The new bags hold 12 cups of popcorn.

After 3 minutes, a submarine had descended to −320 feet. After 8 minutes, the submarine had descended to −420 feet. Assuming a linear function, write an equation in the form d(t)=mt+b that shows the depth, d(t), after t minutes.

Answers

Answer:

  d(t) = -20t -260

Step-by-step explanation:

We are given two points ...

  (t, d) = (3, -320) and (8, -420)

The 2-point form of the equation of a line can be useful when 2 points are given.

  y = (y2 -y1)/(x2 -x1)(x -x1) +y1

Substituting the given points, we have ...

  d(t) = (-420 -(-320))/(8 -3)(t -3) -320

  d(t) = -20(t -3) -320

  d(t) = -20t -260

simplify [tex]\sqrt{.49x^{18} }[/tex] where x<0

Answers

Answer:

-7x⁹

Step-by-step explanation:

[tex]\sqrt{49x^{18}} = \sqrt{(7^2)(x^9)^2} =\sqrt{(7x^{9})^2} = |7x^9|[/tex]

Since  x < 0 ,then |7x⁹| = -7x⁹

Express the inequality x≤−0.12 using interval notation.

Answers

Answer:

(0.12,∞)

Step-by-step explanation:

The inequality x>0.12 means "all numbers greater than 0.12." There is no upper end to the solution to this inequality. In interval notation, we express x>0.12 as (0.12,∞). Notice that the parenthesis symbol shows that the endpoint of the inequality, 0.12, is not included.

The inequality x ≤ - 0.12 is written in interval notation as,

⇒ x ∈ (- ∞, - 0.12 ]

What is Inequality?

A relation by which we can compare two or more mathematical expression is called an inequality.

Given that;

The inequality is,

⇒ x ≤ - 0.12

Now, We can write the inequality in interval notation as;

⇒ x ≤ - 0.12

⇒ x ∈ (- ∞, - 0.12 ]

Learn more about the inequality visit:

https://brainly.com/question/25944814

#SPJ2

Consider the triangle.
Which statement is true about the lengths of the sides?
45°
Each side has a different length.
Two sides have the same length, which is less than the
length of the third side.
D. The three sides have the same length.
D. The sum of the lengths of two sides is equal to the
length of the third side.
45°

Answers

Answer:

Assuming this is a 45 - 45 - 90 right triangle, The answer would be B) Two sides have the same length, which is less than the length of the third side

Hence statements stated true false along with reason

What is triangle?

A triangle is a three-sided polygon with three edges and three vertices in geometry. The sum of a triangle's interior angles equals 180 degrees is the most significant feature of a triangle.

Triangle of Isosceles: Triangle with Obtuse Angle

Triangle Inequality in the Scalene Triangle

Triangle Surface Area: Triangle with Sharp Angle

Triangle with Right Angles: Triangle Form of Pascal...

How to solve?

Given a triangle and statements related to it let's check

Each side has a different length.

-This statement can't always be true as in equilateral triangle all sides are equal

Two sides have the same length, which is less than the

length of the third side.

-this statement is correct but when it is a right angles triangle where other two anges are 45degrees each then this can't be true.

The three sides have the same length.

-this statement is true for equilateral triangle

D. The sum of the lengths of two sides is equal to the

length of the third side.-

-This statement is true when we talk of only right angled triangle

Learn more about triangleshttps://brainly.com/question/17335144

#SPJ2

A 5000​-seat theater has tickets for sale at ​$28 and ​$40. How many tickets should be sold at each price for a sellout performance to generate a total revenue of ​$153 comma 200​?

Answers

Answer:

Let's denote:

x: number of ticket 28$

y: number of ticket 40$

Then, we have:

x + y =5000

28x + 40y = 153200

=> 28(5000 - y) + 40y = 153200

=> 12y = 153200 - 140000

=> 12y =13200

=> y = 1100 (ticket 40$)

=> x = 5000 - 1100 = 3900 (ticket 28$)

log 10(x + 3) – log 10(x-3) = 1

Answers

Answer:33/9

Step-by-step explanation:

Log10(x+3)-Log10(x-3)=1

Log10((x+3)/(x-3))=1

(x+3)/(x-3)=10^1

(x+3)/(x-3)=10

Cross multiply

x+3=10(x-3)

Open brackets

x+3=10x-30

Collect like terms

10x-x=30+3

9x=33

Divide both sides by 9

9x/9=33/9

x=33/9

In the right triangle shown DF=EF=3. How long is DE?

Answers

Answer:

4.24

Step-by-step explanation:

To solve this, use the Pythagorean therom. A^2 + b^2 = C^2

in this case a = 3  and b = 3

so 9 + 9 =  sqrt 18  

4.24

Answer:3√(2)

Step-by-step explanation:

DF=3

EF=3

DE=√(3^2 + 3^2)

DE=√(3x3 + 3x3)

DE=√(9+9)

DE=√(18)

DE=√(2 x 9)

DE=√(2) x √(9)

DE=√(2) x 3

DE=3√(2)

Other Questions
What is the value of x, if the volume of the cone is 12 pi m^3?4 m5 m6 m 10 m -4x-y=24Whats the missing value in the solution to the equation.(__,8) More treaties are entered into than executive agreements. True or false? Abel, a Certified Fraud Examiner (CFE), conducted an interview of Baker, the controller of the ABC Company. Abel asked the following question: "Since you were here when the controls were developed, can you tell me how they came about?" This kind of question is called a ____________________. What is the value of x g science is strictly limited to the study of natural phenomena (things that result as the outcome of natural laws like the speed of light. What is an example of a question that scientific studies cannot address? Question 3 options: 1) What is the purpose of life? 2) Where did an important battle take place? 3) What is the mean flight speed velocity of a sparrow? 4) How much energy is stored in a particular kind of covalent what type of reform does "a story of an hour" suggest Which represents the part of the 10-by-10 grid that is shaded The _____________ Branch of government has the most contact with the residents of Texas.a.Executiveb.Legislativec.Judiciald.all three have equal contact Describe the pattern of the number sequence 1, 9, 17, 25, 33, .... by using algebraic expressions A flask containing 500 liters of hydrogen gas was collected under a pressure of 980mmHg. What will be the volume if the pressure is changed to 900mmHg? IM DOIGN A MATH TEST AND I NEED HELP RN In Circle P, DB and AC are diameters. The measure of angle APD is 50 degrees and PB = 9 cm.Find the length of arc DC. How many servings ofcups are in 5 cups of ice cream? Larry, Barry, Terry are saving their money for a trampoline that costs $340. The amount of money Larry has saved can be represented by x. The amount of money Barry has saved is 112 times the amount Larry has saved. The amount of money Terry has saved is 113 times the amount Larry has saved. The total amount of money saved by Larry, Barry, and Terry is $322.What is x, the amount of money saved by Larry? x2 +8x+16=0 in factoring algorithm A sphere with a diameter of 8 inches. What is the surface area of the sphere shown? 16Pi in2 32Pi in2 64Pi in2 256Pi in2 El coeficiente de variacin de la resistencia con la temperatura del carbn es -0.0005/c.Si la resistencia de una resistencia de carbn es de 1000 a0c , determine su resistencia a 120c Find the volume of the following solid figure. Use = 3.14.V = 4/31tr3. A sphere has a radius of 4.5 inches.Volume (to the nearest tenth) =cubic inches.381.556.584.8 Find the least number which must be added to each of the following numbers to make them a perfect square. Also find the square root of the perfect square number so obtained: a) 5678 Need help with Math question